M ATH 140 A - HW 3 S OLUTIONS
Problem 1 (WR Ch 2 #12). Let K ⊂ R1 consist of 0 and the numbers 1/n, for n = 1, 2, 3, . . .. Prove
that K is compact directly from the definition (without using the Heine-Borel theorem).
Solution. Let {G α } be any open cover of K , which means each G α is an open set and together their
S
union α G α contains K . In order to prove K is compact, we must show there is a finite subcover.
Since 0 ∈ K ⊂ α G α , there is some α0 such that 0 ∈ G α0 . Now we know G α0 is an open set, and
S
an open set must contain a neighborhood of each of its points. Since 0 ∈ G α0 , there is some r > 0
such that Nr (0) ⊂ G α0 . Let N be the smallest integer that is greater than 1/r , which means N > r1 ,
1
or equivalently, r > N. Now notice that
1 1 1
r> > > > ··· ,
N N +1 N +2
or more formally,
r > d (0, N1 ) > d (0, N1+1 ) > d (0, N1+2 ) > · · · ,
1 1 1
which means that the points N , N +1 , N +2 , . . . are all in Nr (0), which is contained in G α0 . So we
have one of the open sets that contains all but a finite number of points of K . The only points
left to cover are 1, 21 , 13 , . . . , N1−1 . Since 1 is in K ⊂ α G α , there must be some α1 such that 1 ∈ G α1 .
S
Repeating this for the rest of the points left, we have sets G α2 ,G α3 , . . . ,G αN −1 containing the points
1 1 1
2 , 3 , . . . , N −1 respectively. Therefore,
N[
−1
K⊂ G αn ,
n=0
and we have found a finite subcover. Hence, K is compact.
Problem 2 (WR Ch 2 #13). Construct a compact set of real numbers whose limit points form a
countable set.
Solution. Let ½ µ ¶¯ ¾
1 1 ¯¯
E= 1 − m, n ∈ N .
2m n ¯
This is plotted below,
A more illustrative plot follows, with the x-axis representing points of E and the y-axis represent-
ing different values of m to visually separate out different groups of points.
1
1
Consider the points of the form p = 2m with m ∈ N. Any neighborhood of one of these points
1 1
of radius r > 0 will also contain the point q = 2m (1 − n ) where we choose the positive integer n
1
such that n <2 m
r , so that |p − q| = | 21m − 1 1 1
2m (1 − n )| = | 2m n | < r . Since q 6= p and q ∈ E , that means
p is a limit point, and thus E has at least a countably infinite number of limit points.
The fact that E is compact comes from its being closed (since it contains its limit points) and
bounded (since each point of E is contained in [0, 21 ]).
Problem 3 (WR Ch 2 #22). A metric space is called separable if it contains a countable dense
subset. Show that Rk is separable.
Solution. We claim that Qk is dense in Rk . To show this, let p = (p 1 , . . . , p k ) ∈ Rk . In order to show
Qk is dense, we need to show that p ∈ Qk or that p is a limit point of Qk . So if p ∈ Qk , we’re done.
If p ∉ Qk , we want to show that p is a limit point. Let Nr (p) with r > 0 be a neighborhood of p. Let
p
δ = r / n, and since Q is dense in R, we can find some q i 6= p i in Nδ (p i ) for i = 1, . . . , k. Then for
q = (q 1 , . . . , q k ) we have
s
r2 r2
q
d (p, q) = (p 1 − q 1 )2 + · · · + (p k − q k )2 < +···+ = r,
n n
so that q ∈ Nr (p), and thus p is a limit point.
Thus Qk is dense in Rk , and it is countable by Theorem 2.13, so Rk is separable.
Problem 4 (WR Ch 2 #24). Let X be a metric space in which every infinite subset has a limit point.
Prove that X is separable.
Solution. Fix δ > 0 and pick x 1 ∈ X . Having chosen x 1 , . . . , x j ∈ X , choose x j +1 ∈ X , if possible, so
that d (x i , x j +1 ) ≥ δ for i = 1, . . . , j . This is essentially covering X by disjoint d el t a-balls centered
at the points x 1 , x 2 , . . .. If we can do this forever without having any overlap, then the set {x j | j ∈ N}
is an infinite set without a limit point (since we have neighborhoods around each point that don’t
2
contain any other point in the set). But his is a contradiction, so this process must stop after a
finite number of steps, which means X can be covered by finitely many neighborhoods of radius
δ. Since we have proved this for any δ > 0, we can assign δ to be anything greater than zero and
still have the same result.
Let δ = 1. Then there are finitely many points x 1(1) , x 2(1) , . . . , x N
(1)
such that X is covered by δ-
1
neighborhoods centered at those points. Now let δ = 12 . Then there are finitely many points
x 1(2) , x 2(2) , . . . , x N
(2)
such that X is covered by δ-neighborhoods centered at those points. Repeating
2
this for any n ∈ N we let δ = 1
n and then there are finitely many points x 1(n) , x 2(n) , . . . , x N
(n)
such that
n
X is covered by δ-neighborhoods centered at those points. We now claim that the set
E = {x 1(1) , x 2(1) , . . . , x N
(1)
,
1
x 1(2) , x 2(2) , . . . , x N
(2)
,
2
x 1(3) , x 2(3) , . . . , x N
(3)
,
3
. . .}
is a countable dense subset of X . It’s countable by Theorem 2.12. It’s dense because for any point
1
x ∈ X and any neighborhood Nr (x) for r > 0, we can choose a positive integer n such that n < r,
and then either x = x i(n) for some 1 ≤ i ≤ Nn , or x is in some neighborhood N 1 (x i ) for some
n
1 ≤ i ≤ Nn because we have an open cover. That means that x i ∈ Nr (x) and x i 6= x, meaning x is a
limit point of E .
Problem 5 (WR Ch 2 #25). Prove that every compact metric space K has a countable base, and
that K is therefore separable.
1
Solution. For each n ∈ N, make an open cover of K by neighborhoods of radius n, and we have a
finite subcover by compactness, i.e.
[ N
[
K⊂ N 1 (x) =⇒ ∃ x 1 , . . . , x N ∈ K such that K ⊂ N 1 (x i )
n n
x∈K i =1
Doing this for every n ∈ N, we get a countable union of finite collections of sets, so that by Theorem
2.12, the collection of these sets, call it S, is countable.
We claim that S is a countable base for K , which is defined as a countable collection of open
sets such that for any x ∈ K and any open set G with x ∈ G, there is some V ∈ S such that x ∈ V ⊂ G.
Let x ∈ K and let G be any open set with x ∈ G. Then since G is open, there is some r > 0 such
1
that Nr (x) ⊂ G. Choose n ∈ N such that n < r /2, so that the maximal distance between points in a
1
neighborhood of radius n is r . Then there must be some i such that x ∈ N 1 (x i ) ⊂ Nr (x) because
n
any neighborhood of radius 1/n containing x cannot contain points a distance more than r away.
This shows that S is a countable base.
The second part of the question asks us to show that K is separable. Let {Vn } be our countable
base for K . For each n ∈ N, choose x n ∈ Vn , and let E = {x n |n ∈ N}. We claim that E is a countable
3
dense set, which would show that K is separable. First, note that E is clearly countable. To show
that it’s dense, we need to show that Ē = K . This is equivalent to showing that (Ē )c = ;. Now (Ē )c
is an open set because it’s the complement of a closed set, Ē . If (Ē )c is nonempty, then there is
some x ∈ (Ē )c , which is open, so since {Vn } is a base, there is some n such that x ∈ Vn ⊂ (Ē )c , which
implies that x n ∈ (Ē )c , a contradiction, because xn ∈ E ⇒ x n ∈ Ē ⇒ x n ∉ (Ē )c . Therefore,
(Ē )c = ;, so that Ē = K .
Problem 6 (WR Ch 2 #26). Let X be a metric space in which every infinite subset has a limit point.
Prove that X is compact.
Solution. By exercises 23 and 24, X has a countable base. Thus for any open cover {G α }, we can
write each G α as a union of sets from the countable base, meaning that every open cover has
a countable subcover {G n }. Assume by way of contradiction that no finite subcollection of {G n }
covers X . By setting à !c
n
[
Fn = Gi ,
i =1
we have that F n 6= ; for each n since if that were the case, X = F nc =
Sn
i =1
G i , which means {G 1 , . . . ,G n }
would be a finite subcover. Notice also that F n+1 ⊂ F n , since we are only removing points as n goes
up. Now we also have
à !c à à !!c à !c
∞ ∞ [ n ∞ [ n ∞
= X c = ;,
\ \ [ [
Fn = Gi = Gi = Gi
n=1 n=1 i =1 n=1 i =1 i =1
T
since {G n } is an open cover of X . Thus F n is empty. We now create an infinite set E by choosing
some x n ∈ F n (which is nonempty) for each n ∈ N and letting E = {x n |n ∈ N}. The only way that E
could not be infinite is if some x ∈ E were in an infinite number of sets F n , but that would make
T
F n nonempty. Since E is an infinite set, E has a limit point p (which is given in the beginning
of the problem statement). For each n, all but finitely many points of E lie in F n , so P must be
T
a limit point of F n for all n. But the F n ’s are closed, so p ∈ F n for all n, meaning that F n 6= ;, a
contradiction. Therefore, any open cover of X has a finite subcover, and thus X is compact.
Problem 7 (WR Ch 2 #29). Prove that every open set in R1 is the union of an at most countable
collection of disjoint segments.
Solution. By exercise 22, R1 is separable, and thus has a countable dense set, namely Q.
Let G ⊂ R be any open set. Then Q ∩G is a countable dense set in G by the Archimedean prop-
erty, and since G is open we can choose an open interval around every rational in G. Then G is the
union of that countable collection of intervals. However, we need to find a countable collection of
disjoint intervals. Notice that the union of any intervals which contain the same point is an inter-
val with a lower endpoint equal to the infimum of the lower endpoints of the intervals (possibly
−∞) and with an upper endpoint equal to the supremum of the upper endpoints of the intervals
4
(possibly ∞). We create a new countable collection of intervals whose union is G by the following
procedure.
Take any point in G ∩Q and take the union of all intervals in G that contain it. Call this interval
I 1 . Now take some point in (G \ I 1 ) ∩ Q and take the union of all intervals in G \ I 1 that contain it.
Repeating this process we get a countable collection of disjoint intervals I 1 , I 2 , I 3 , . . ., each of which
is in G and which together cover G.
Problem 8 (WR Ch 2 #30). If Rk = F n , where F n is a closed subset of Rk , then at least one F n
S∞
1
has a nonempty interior.
Sn
Solution. Assume by way of contradiction that each F n has an empty interior. Let Vn = i =1
Fi .
Since F 1 is closed, F 1c is open. If F 1c were empty, then F 1 = Rk , but then F 1◦ 6= ;, so instead F 1c
must be nonempty. Let K 1 be some neighborhood in F 1c such that K 1 ∩ V1 = ; (which we can
do by shrinking the neighborhood if necessary). Now if we have defined K n so that K n ∩ Vn = ;,
we define K n+1 by taking a neighborhood in K n \ F n+1 , which is nonempty because F n+1 has a
nonempty interior. By shrinking if necessary, we can ensure that K n+1 ⊂ K n . Notice again that
K n+1 ∩ Vn+1 = ;. This last property gives us that ∞
T
1 K n is disjoint from every F n . Also, since each
K n is compact and K n+1 ⊂ K n then by Theorem 2.39 we know that ∞
T
1 K n is nonempty. Thus, there
is some point ¶c
∞ µ∞
= (Rk )c = ;,
\ [
x∈ Kn ⊂ Fn
1 1
a contradiction, since the empty set cannot have a point in it.